LSAT and Law School Admissions Forum

Get expert LSAT preparation and law school admissions advice from PowerScore Test Preparation.

 Adam Tyson
PowerScore Staff
  • PowerScore Staff
  • Posts: 5153
  • Joined: Apr 14, 2011
|
#60850
Strong language may sometimes be okay when a principle is involved, hasan, especially if the principle is one that is meant to strengthen, justify, or weaken the argument in the stimulus. Strong language is suspect when we are in a Must Be True paradigm, looking for a principle that follows from the stimulus. Other than that, your analysis looks good! Nice job correcting your error regarding B.
 rishrish
  • Posts: 1
  • Joined: Nov 23, 2019
|
#72230
Why is this question an MBT-PR? Shouldn't it be a strenghth-PR? I thought MBT is if the answer choices conform to the principle stated in the stimulus. This question requires the stimulus to conform to one of the principles in the answer choice.

This is why I chose B. I thought that B strengthens the argument because if the tariffs on the cheaper and better imported fruit was imposed in order to save the domestic fruit growers, the government is clearly putting the interest of the producer over the consumer (since it is in the interest of the consumer to buy the cheaper and better fruit), and B is saying that should always happen. What am I doing wrong???
 Jeremy Press
PowerScore Staff
  • PowerScore Staff
  • Posts: 1000
  • Joined: Jun 12, 2017
|
#72262
Hi rishrish,

There's definitely somewhat trickier wording in this question stem, but I agree with the PowerScore consensus here that this is a Must Be True-Principle question stem.

One thing a Strengthen-Principle question stem will always have (that this stem is lacking) is some indication that the principle is to be taken as valid or true. That's a characteristic of all Family 2 (and Family 3, for that matter) questions. A Strengthen-Principle question stem will also contain language indicating the answer is to be used to provide some additional assistance or support for the argument in the stimulus.

When we see the language of a Principle question stem asking us to judge the level of conformity (i.e. the "match") between the conclusion in the stimulus and the principles expressed in the answer choices, we're being asked to perform a classic "Must Be True" task. The only way to judge the conformity is to stick closely to the reasoning in the stimulus. In other words, the correct principle shouldn't stray beyond the boundaries that the stimulus reasoning implies. Here, the stimulus reasoning addresses only one specific economic/social scenario, so we can't be sure there is "conformity" (using just that one scenario) with a principle expressed in the strong terms of answer choice B (using the "always" language that others in this thread have noted).

I hope this helps!

Jeremy

Get the most out of your LSAT Prep Plus subscription.

Analyze and track your performance with our Testing and Analytics Package.